- PowerScore Staff
- Posts: 5972
- Joined: Mar 25, 2011
- Mon Jul 11, 2016 11:48 am
#26963
Complete Question Explanation
The correct answer choice is (A)
In a fortuitous circumstance, this question poses a condition that is met by the hypothetical created in question #3. Thus, we can use that hypothetical to eliminate certain answers such as (B) and (D). However, That said, let us examine this problem as if we did not realize that the hypothetical from question #3 applied to question #4.
If T and X are assigned to the same row (and next to each other), then the other two variables from the S, T, U, X group must be together. Thus, S and U must be in the same row as each other, and we can conclude that S and U must be in the last row (remember, neither can be beside N) and that therefore T and X must be in the front row. Further, from the question stem, T must be seated next to X and another traveler, and so T must be in seat 2. This information is sufficient to prove answer choice (A) correct.
Answer choice (B) is incorrect because if T is assigned to the last row with E, then S and U would be assigned to the first row with N, creating a violation of the last rule.
Answer choice (C) is incorrect because although this could occur in the last row, it does not have to occur. As this is a Must Be True question, this answer is therefore incorrect.
Answer choice (D) is incorrect because O and T must be in different rows, and thus they can never be seated beside each other.
Answer choice (E) is incorrect because although this could occur in the last row, it does not have to occur. As this is a Must Be True question, this answer is therefore incorrect.
The correct answer choice is (A)
In a fortuitous circumstance, this question poses a condition that is met by the hypothetical created in question #3. Thus, we can use that hypothetical to eliminate certain answers such as (B) and (D). However, That said, let us examine this problem as if we did not realize that the hypothetical from question #3 applied to question #4.
If T and X are assigned to the same row (and next to each other), then the other two variables from the S, T, U, X group must be together. Thus, S and U must be in the same row as each other, and we can conclude that S and U must be in the last row (remember, neither can be beside N) and that therefore T and X must be in the front row. Further, from the question stem, T must be seated next to X and another traveler, and so T must be in seat 2. This information is sufficient to prove answer choice (A) correct.
Answer choice (B) is incorrect because if T is assigned to the last row with E, then S and U would be assigned to the first row with N, creating a violation of the last rule.
Answer choice (C) is incorrect because although this could occur in the last row, it does not have to occur. As this is a Must Be True question, this answer is therefore incorrect.
Answer choice (D) is incorrect because O and T must be in different rows, and thus they can never be seated beside each other.
Answer choice (E) is incorrect because although this could occur in the last row, it does not have to occur. As this is a Must Be True question, this answer is therefore incorrect.
Dave Killoran
PowerScore Test Preparation
Follow me on X/Twitter at http://twitter.com/DaveKilloran
My LSAT Articles: http://blog.powerscore.com/lsat/author/dave-killoran
PowerScore Podcast: http://www.powerscore.com/lsat/podcast/
PowerScore Test Preparation
Follow me on X/Twitter at http://twitter.com/DaveKilloran
My LSAT Articles: http://blog.powerscore.com/lsat/author/dave-killoran
PowerScore Podcast: http://www.powerscore.com/lsat/podcast/